Exercice 04-03
Calculer les limites \(n\to \infty\) des suites ci-dessous, qui sont toutes des indéterminations ''\(1^\infty\)''.
  1. \(x_n=\bigl(1+\tfrac1{n^2}\bigr)^{n^2}\)
  2. \(y_n= \bigl(1+\tfrac1{n}\bigr)^{n^2}\)
  3. \(z_n = \bigl(1+\tfrac1{n^2}\bigr)^n\)
Au cours, on a étudié la limite fondamentale \[ \lim_{n\to \infty} \bigl(1+\tfrac1n\bigr)^n=e=2.718\dots\,. \] Donc si on pose \(e_n=(1+\frac1n)^n\), on sait que \(e_n\to e\).

On pourra donc essayer de relier chacune des suites \(x_n\), \(y_n\), \(z_n\) à \(e_n\).
Soit \(e_n=(1+\frac1n)^n\). Rappelons ce que signifie ''\(e_n\to e\)'': pour tout \(\varepsilon>0\), il existe \(N\in \mathbb{N}\) tel que \[|e_n-e|\leqslant \varepsilon\qquad \forall n\geqslant N\,.\]
  1. Montrons que \(x_n\to e\). Remarquons que \(x_n=(1+\frac{1}{n^2})^{n^2}=e_{n^2}\). Fixons donc \(\varepsilon>0\), et considérons le \(N\) ci-dessus. Si on prend \(n\geqslant [\sqrt{N}]+1\), alors \(n^2\geqslant N\), et donc \(|e_{n^2}-e|\leqslant \varepsilon\). Ceci montre que \(x_n\to e\).

    Remarque: \(x_n=e_{n^2}\) est ce qu'on appellera plus tard une sous-suite de \((e_n)\).

  2. Commençons par écrire \[ y_n= \Bigl(1+\frac1{n}\Bigr)^{n^2}= \Bigl[\Bigl(1+\frac1{n}\Bigr)^{n}\Bigr]^{n}={e_n}^n\,. \] Comme \(e_n=(1+\frac1{n})^{n}\to e\), et comme \(e>2\), il existe \(N\) tel que \(e_n\geqslant 2\) pour tout \(n\geqslant N\). On a donc aussi \(y_n={e_n}^n\geqslant 2^n\) pour tout \(n\geqslant N\). Comme \(2^n\to \infty\), on en conclut que \(y_n\to \infty\).

    Remarque: Si on sait que \(y_n={e_n}^n\), et que \(e_n\to e\gt 2\), on a peut-être envie d'écrire quelque chose comme ''\(y_n\to e^\infty=\infty\)'', ce qui doit convaincre que la limite de \(y_n\) est \(+\infty\) mais ne fait malheureusement aucun sens. Donc pour rendre l'argument rigoureux on a simplement utilisé le fait que \(e\gt 2\) pour construire un ''chien méchant'': \(y_n\geqslant 2^n\) pour \(n\) grand.

  3. Remarquons d'abord que \(z_n={x_n}^{1/n}\). On a montré plus haut que \(x_n\to e\). Soit donc \(0<\varepsilon< e\) et soit \(N\) tel que pour tout \(n\geqslant N\) on ait \(|x_n-e|\leqslant \varepsilon\), c'est-à-dire \(e-\varepsilon\leqslant x_n\leqslant e+\varepsilon\). On peut alors écrire, lorsque \(n\geqslant N\), que \[ (e-\varepsilon)^{1/n} \leqslant z_n \leqslant (e+\varepsilon)^{1/n} \] Mais puisque \((e\pm \varepsilon)^{1/n}\to 1\), on a que \(z_n\to 1\).

    Remarque: Le fait que \(c^{1/n}\to 1\) pour toute constante \(c\gt 0\) vient de \[\begin{aligned} c^{1/n} &=\left(\exp(\log(c))\right)^{1/n}\\ &=\exp\left(\frac{\log(c)}{n}\right) \to \exp (0)=1 \end{aligned}\]